LSAT 90 – Section 4 – Question 24

You need a full course to see this video. Enroll now and get started in less than a minute.

Target time: 1:34

This is question data from the 7Sage LSAT Scorer. You can score your LSATs, track your results, and analyze your performance with pretty charts and vital statistics - all with a Free Account ← sign up in less than 10 seconds

Question
QuickView
Type Tags Answer
Choices
Curve Question
Difficulty
Psg/Game/S
Difficulty
Explanation
PT90 S4 Q24
+LR
Weaken +Weak
A
9%
154
B
14%
156
C
8%
155
D
60%
164
E
8%
160
147
157
166
+Harder 148.293 +SubsectionMedium

This is a Weakening question.

The consumer advocate says some agricultural crops are now being genetically engineered to produce important pharmaceuticals. That's context. However, this development raises the possibility that the drugs will end up in the general food supply, since (premise indicator) if pollen from a drug-producing crop drifts into a nearby field in which an ordinary, non-drug-producing crop of the same species is being grown, the pollen could fertilize that crop and turn it into a drug-producing crop as well.

This is a pretty cookie-cutter structure. We get the context, conclusion, and the premises. The context is that there are some crops, for example, watermelons, being genetically engineered to produce drugs. And the conclusion is that this raises the possibility that these drugs might end up in the general food supply.

Why is this so? Say a farmer is growing regular watermelons for food, and in the next field a pharmaceutical company is growing genetically engineered watermelons for drugs. The consumer advocate is saying that if pollen from this genetically engineered watermelon drifts into our farmer’s field, then it could fertilize the regular watermelons and turn them into drug-producing watermelons. And the farmer might not know this and just ship the watermelons off, which would lead to drugs being in the food supply.

If you do not see the gap in the reasoning already, that's fine. I usually cannot predict the correct answer in Weakening questions unless they are really obvious, so I am going to use process of elimination.

Answer Choice (A) says that as far as scientists know, none of the pharmaceuticals produced by genetically engineered crops would present any danger to public health if they were present in the general food supply. If you picked (A), the LSAT writers might have successfully distracted you from your objective.

(A) is saying that even if the conclusion was true, there would not be a public health danger. Whew, crisis averted! This is not how you weaken an argument. To weaken an argument, you have to say that even if the premises are true, the conclusion might not be supported, not that even if the conclusion were true, there wouldn't be a danger to public health. We are not trying to accept the conclusion and convince the consumer advocate that this is not a big deal.

Answer Choice (C) is similar. (C) says the genetically engineered crops that produce pharmaceuticals are not among the crop species that comprise the largest portions of the general food supply. What does this even mean? First, imagine the set of general food supply. Next, what "crop species comprise the largest portions" of that set? Probably the staple crops like corn, wheat, potatoes, etc. (C) is saying that the genetically engineered crops are not in this set of staple crops. Oh, that's good. I mean, we already used watermelons for illustration so it already wasn't in the set. But wait, does that matter?

Like (A), (C) is trying to make you accept the conclusion and convince you that it is not a big deal. It is fine because genetically engineered crops only comprise a small portion of the general food supply. I concede that both (A) and (C) might be latching on to and attempting to mitigate the concern which motivated the consumer advocate in the first place, but neither is weakening her argument.

Answer Choice (B) says if pollen from a genetically engineered crop is prevented from drifting into fields in which ordinary crops of the same species are being grown, then there is no risk of the pollen fertilizing the latter crops. Sure, but the consumer advocate is saying that if the pollen drifts, then the resulting crop will be a drug-producing crop. (B) just says if pollen does not drift, then there is no danger. That doesn't work. If the premise places us in a hypothetical world where pollen drifts, refusing to be in that world won't do anything to weaken the argument.

Answer Choice (E) says that if pollen from a drug-producing crop turned an ordinary crop of the same species into a drug-producing crop, it would be possible for scientists to identify the latter crop. In order for (E) to weaken, you need to make a pretty big assumption about the consequences of the scientists' capacity to identify the latter (drug-producing) crop. What will they do? Will they act to stop the accidental crop from entering the general food supply? Will they throw up their hands and say, "Oh well"? (E) is silent.

Also notice that (E) does not tell you anything about when the scientists are able to identify this. It just says it is possible. But surely timing matters. Imagine a scientist is enjoying her watermelon and notices that it tastes kind of funny, like the watermelon she produced in a lab. She now knows something bad happened, but drugs are now already in the food supply.

Correct Answer Choice (D) says that in crops genetically engineered to produce pharmaceuticals, the drugs are not present in any part of the plant used for food in ordinary crops of the same species. This is a classically correct weakening answer choice. It reveals a naive assumption we made, which was that it is the flesh of the watermelon that would contain the drugs.

(D) says this is not the case. (D) says that it is the leaves or the stems of the watermelons that contain the drugs. If (D) were true, then even if food-producing watermelon crop turned into drug-producing watermelon crop, the drugs would not end up in the general food supply.

Take PrepTest

Review Results

Leave a Reply